Đến nội dung

redfox nội dung

Có 96 mục bởi redfox (Tìm giới hạn từ 30-03-2020)



Sắp theo                Sắp xếp  

#658248 Đề thi chọn đội tuyển quốc gia tỉnh Quảng Bình

Đã gửi bởi redfox on 17-10-2016 - 22:31 trong Thi HSG cấp Tỉnh, Thành phố. Olympic 30-4. Đề thi và kiểm tra đội tuyển các cấp.

Bài 4

Tính tổng các cặp số:

-Có $2k$ số phân biệt nên tổng lớn hơn $\frac{2k(2k+1)}{2}$.

-Tổng các cặp số là $k$ số phân biệt nên tổng nhỏ hơn $nk-\frac{k(k-1)}{2}$.

Vậy $\frac{2k(2k+1)}{2}\leq nk-\frac{k(k-1)}{2}\Rightarrow k\leq \frac{2n-1}{5}\Rightarrow k\leq \left \lfloor \frac{2n-1}{5} \right \rfloor$.

Xét $n=5m+3$. Ta chọn $m$ cặp ${2i,3m+2-i},1\leq i\leq m$ và $m+1$ cặp ${2i-1,4m+3-1},1\leq i\leq m+1$ thỏa mãn.

$n=5m+4,5$ làm như trên, $n=5m+1,2$ bỏ cặp ${3m+2,2m+1}$.

Từ đó ta có $k= \left \lfloor \frac{2n-1}{5} \right \rfloor$.




#654573 Đề thi chọn đội tuyển quốc gia tỉnh Quảng Bình

Đã gửi bởi redfox on 17-09-2016 - 22:33 trong Thi HSG cấp Tỉnh, Thành phố. Olympic 30-4. Đề thi và kiểm tra đội tuyển các cấp.

Giả sử $\exists z,f(z)>z$. Đặt  $\alpha =f(z)-1,M=max(f(x)\mid 1\leq x\leq \alpha )$.

Với $x>\alpha$,ta có $z+f(x)\geq f(x+\alpha )+1\Rightarrow f(x+n\alpha )\leq f(x)+n(z -1)$

Đặt $g(x)\equiv x(mod \alpha )$ ($1\leq g(x)\leq \alpha$).

Khi đó $f(x)=f(\frac{x-g(x)}{\alpha }\alpha +g(x))\leq f(g(x))+\frac{(x-g(x))(z-1)}{\alpha }$. Thay vào $f(f(x))=x$, chọn $x$ đủ lớn dễ thấy mâu thuẫn.

Vậy $f(x)\leq x\Rightarrow f(f(x))\leq x= f(f(x))\Rightarrow f(x)=x$.

Sai phân có $f(x)$ trong mối quan hệ đâu mà áp dụng, chính bạn nói cần $x,f(x),f(f(x))$ 




#667570 Đề cử Thành viên nổi bật 2016

Đã gửi bởi redfox on 08-01-2017 - 08:52 trong Thông báo tổng quan

1. Tên nick ứng viên: bangbang1412, baopbc

2. Thành tích nổi bật:

-baopbc: Tích cực tham gia Mỗi tuần 1 bài toán hình học

-bangbang1412: Thảo luận sôi nổi trên topic tôpô

3. Ghi chú: Không có




#658244 Đề thi chọn đội tuyển quốc gia THPT chuyên KHTN - ĐHQG Hà Nội vòng 1 năm 2016

Đã gửi bởi redfox on 17-10-2016 - 22:18 trong Thi HSG cấp Tỉnh, Thành phố. Olympic 30-4. Đề thi và kiểm tra đội tuyển các cấp.

Bài 2: Ta cũng có thể xài đơn biến. Cuốn sách sai vị trí ta điền số $1$, đúng vị trí ta điền số $0$. Xét số nhị phân được tạo thành.

Sau khi chuyển $1$ cuốn sách về đúng vị trí, chữ số hàng đó chuyển từ $1$ đến $0$, các chữ số trước nó không thay đổi. Vậy số nhị phân luôn giảm.

Từ trạng thái ban đầu, số không lớn hơn $2^n-1$, đến trạng thái cuối, số đó là $0$. Vậy cần ít hơn $2^n$ bước.

(Q.E.D)




#688186 58th IMO 2017

Đã gửi bởi redfox on 20-07-2017 - 20:21 trong Thi HSG Quốc gia và Quốc tế

Câu 6: Ta coi $gcd(0,1)=1$. Gọi các điểm của $S$ là $(x_1,y_1),...,(x_k,y_k)$.

1: $gcd(a_n,x_1...x_k)=1$ nếu tồn tại đa thức thỏa mãn (chú ý $a_ny^n\equiv 1(mod\; x_i)$).

2: Có thể coi $n\geq k$ và $a_n=1$.

Ta có đa thức $P$ thỏa mãn đề bài thì $P^p$ ($p$ nguyên dương) cũng thỏa hay $n$ thỏa mãn thì $pn$ cũng thỏa, ta chọn $p$ đủ lớn sao cho $pn\geq k$

Ta có đa thức $P$ có bậc $n\geq k$ thỏa mãn đề bài thì $P^p-qy^{pn-k}(x_1y-y_1x)...(x_ky-y_kx)$ ($p,q$ nguyên dương) cũng thỏa. Hệ số của $y^{np}$ là $a_n^p-qx_1...x_k$. Theo 1 và định lí Euler, tồn tại $p,q$ sao cho hệ số trên bằng $1$.

3: Có thể coi $(0,1)\in S$.

Ta có đa thức $P(x,y)$ thỏa mãn với $S$ khi và chỉ khi đa thức $P(x+y,y)$ thỏa mãn với $S'=\left \{(x_1-y_1,y_1);...;(x_n-y_n,y_n) \right \}$, các cặp số của $S'$ vẫn gồm các điểm nguyên thủy phân biệt. Bằng cách làm bước trên, thay đổi vai trò của $x,y$ và áp dụng thuật toán Euclid, ta có thể đưa $(x_1,y_1)$ về $(0,1)$.

Ta sẽ chứng minh bằng quy nạp với $k$:

$k=1$ dễ tồn tại đa thức thỏa mãn (định lý Bezout).

Giả sử $k-1$ đúng, áp dụng 3, ta coi $(0,1)\in S$. Theo giả thiết quy nạp, tồn tại đa thức $P$ thỏa mãn với $k-1$ cặp số còn lại. Áp dụng 2, ta coi hệ số $y^n$ bằng $1$, dễ thấy đa thức này cũng đúng cho $(0,1)$ hay cho $S$.

(Q.E.D)




#662383 Đề thi HSG toán 10 trường THPT chuyên KHTN (lần 1)

Đã gửi bởi redfox on 19-11-2016 - 00:03 trong Thi HSG cấp Tỉnh, Thành phố. Olympic 30-4. Đề thi và kiểm tra đội tuyển các cấp.

Câu IV:

Chia $(1;2);(3;4);...;(2k-1;2k);...(4029;4030)$, ta chỉ cần xét khoảng $[2015^2;2015^2+2015]$.

Chia $(1;2016);(2;2017);...;(k;2015+k);,,,;(2015;4030)$, ta chỉ cần xét hai số $2015^2;2015^2+2015$(xét đồng dư modulo $2015$).

Chia $(1;4030);(2;4);(3;5);...(4k-2;4k);(4k-1;4k+1);...(4026;4028);(4027;4029)$, ta được các số chẵn nhỏ hơn $2015^2+2015$ và các số lẻ lớn hơn $2015^2$, loại được hai số trên.

(Q.E.D)




#658253 Đề thi chọn ĐT dự thi HSGQG Đà Nẵng, 2016-2017

Đã gửi bởi redfox on 17-10-2016 - 22:47 trong Thi HSG cấp Tỉnh, Thành phố. Olympic 30-4. Đề thi và kiểm tra đội tuyển các cấp.

Bài 2

Ta có với mọi $n\leq 2015$, $2017\mid \sum_{k=1}^{2016}k^n$( lời giải có thể tham khảo tại đây http://diendantoanho...-kiểm-tra-số-2/)

Giả sử bậc của $f$ nhỏ hơn $2015$. Ta có $f(x)\equiv 0,1(mod 2017)$. Cộng $f(x)$ từ $0$ đến $2016$, từ trên ta có $\sum_{k=0}^{2016}f(x)\equiv 0(mod 2017)$.

Vậy $f(x)\equiv 0(mod 2017)$ với $2017$ giá trị của $x$ nên $f\equiv 0$ (vô lí).

Vậy bậc của $f$ lớn hơn hoặc bằng $2016$. Với $n=2016$, lấy $f(x)=x^{2016}$ thỏa do Fermat. Vậy $n=2016$.




#655360 Đề thi chọn ĐT dự thi HSGQG Đà Nẵng, 2016-2017

Đã gửi bởi redfox on 24-09-2016 - 15:50 trong Thi HSG cấp Tỉnh, Thành phố. Olympic 30-4. Đề thi và kiểm tra đội tuyển các cấp.

Cách giải này chuẩn rồi, nhưng bạn còn thiếu phần chỉ ra rằng có thể đánh dấu để số ở hai miền chung cạnh đều trái dấu!

Với $n=2$, ta có cách đánh dấu như vậy.

Giả sử có cách đánh dấu với $n$ đường thẳng. Với $n+1$ đường thẳng, ta bỏ một đường thẳng đi và đánh dấu. Sau đó đưa đường thẳng đó trở lại và đổi dấu ở các miền ở một trong hai nửa mặt phẳng. Các miền không bị chia cắt thoả mãn. Hai miền bị chia từ một miền bị đổi dấu một miền, trở thành trái dấu thoả mãn.

Còn về dấu ở hai miền chung góc, hai miền đó cùng trái dấu với một miền nên cùng dấu.




#655302 Đề thi chọn ĐT dự thi HSGQG Đà Nẵng, 2016-2017

Đã gửi bởi redfox on 23-09-2016 - 22:52 trong Thi HSG cấp Tỉnh, Thành phố. Olympic 30-4. Đề thi và kiểm tra đội tuyển các cấp.

Bài 4

Đánh các số vào các miền sao cho trị tuyệt đối bằng số đỉnh của miền, hai số ở hai miền chung cạnh trái dấu, chung đỉnh cùng dấu.

(i) hiển nhiên thoả mãn.

(ii) vì có $n$ đường thẳng nên số đỉnh mỗi miền không lớn hơn $n$, thoả mãn.

(iii) ta chia đều số trong các miền các số $1,-1$ cho các góc của nó. Xét đường thẳng bất kì chia mặt phẳng thành hai miền, tổng các số được đánh trên $2$ hoặc $4$ góc của mỗi điểm trên nửa mặt phẳng đều bằng $0$. Cộng các số trên các đỉnh lại bằng $0$, thoả mãn.

(Q.E.D)




#698962 Đề cử Thành viên nổi bật 2017

Đã gửi bởi redfox on 26-12-2017 - 21:44 trong Thông báo tổng quan

1. Tên Nick ứng viên: vutuanhien

2. Thành tích (đóng góp) nổi bật: Là thành viên thảo luận sôi nổi trên nhiều topic Toán Đại cương




#650813 Đề kiểm tra đội tuyển toán Chuyên Bảo Lộc (Lâm Đồng)

Đã gửi bởi redfox on 22-08-2016 - 17:41 trong Thi HSG cấp Tỉnh, Thành phố. Olympic 30-4. Đề thi và kiểm tra đội tuyển các cấp.

Câu 5(lần 3)

a) Giả sử có cách sắp như vậy. Vì bảng có $8$ điểm nên mỗi đường có số lẻ viên sỏi.

Xét $2$ cột hai bên, $2$ đường chéo ,$1$ hàng ở giữa. Ta xét tổng các viên sỏi của các đường (kể cả lặp).

Mỗi ô trừ ô giữa lặp $2$ lần nên tổng là số chẵn.

Có $5$ đường, mỗi đường có số lẻ viên sỏi nên tổng là số lẻ.

Mâu thuẫn này dẫn đến không tồn tại cách sắp như vậy.

b) Đổi trạng thái một ô góc cố định (không sỏi thành có sỏi và ngược lại). Khi đó tính chẵn lẻ của số viên sỏi của $3$ đường đi qua ô góc này thay đổi nên tính chẵn lẻ điểm của bảng thay đổi. Vậy bằng cách làm này ta thành lập một song ánh giữa bảng có điểm số chẵn và bảng có điểm số lẻ hay số cách đặt sỏi bằng nhau.




#658765 Chứng minh rằng tồn tại $f(A)=H \setminus B,g(B)= H \setmi...

Đã gửi bởi redfox on 22-10-2016 - 11:36 trong Các dạng toán khác

Ta có $f(H)\subset H$ (hiển nhiên), bằng quy nạp $f^{n+1}(H)\subset f^n(H)$. Vậy $f^n(H)$ có giới hạn thỏa $f(X)=X$.

Không biết xài mấy từ này có ổn không.




#658791 Chứng minh rằng tồn tại $f(A)=H \setminus B,g(B)= H \setmi...

Đã gửi bởi redfox on 22-10-2016 - 17:25 trong Các dạng toán khác

Bổ đề: Nếu $f$ tăng trên $\rho (H)$ thì $f$ tồn tại một điểm bất động

Xét dãy tập hợp $X_0=H, X_{n+1}=f(X_n)$. Ta sẽ chứng minh bằng quy nạp $X_{n+1} \subset X_n$.

Với $n=0$, ta có $f(H)\subset H$ vì $f(H)$ thuộc $\rho (H)$ nên là tập con của $H$.

Giả sử $X_{n+1}\subset X_n$, theo định nghĩa hàm tăng, $f(X_{n+1})\subset f(X_n)$ hay $X_{n+2}\subset X_{n+1}$

Vậy $X_{n}$ dần tiến về tập $X\subset H$ (đoạn này thấy không ổn nhưng có vẻ đúng với tập hữu hạn). Ta có $f(X)=X$

Rồi làm như phần chứng minh trên.




#658753 Chứng minh rằng tồn tại $f(A)=H \setminus B,g(B)= H \setmi...

Đã gửi bởi redfox on 22-10-2016 - 09:22 trong Các dạng toán khác

$X=\lim_{n\rightarrow \infty }f^n(H)$




#658654 Chứng minh rằng tồn tại $f(A)=H \setminus B,g(B)= H \setmi...

Đã gửi bởi redfox on 21-10-2016 - 10:21 trong Các dạng toán khác

Ý tưởng của em là chỉ cần xét khoảng $[0;1)$. Xét tập $D_n=\left \{ [0;\frac{1}{2^n});...[1-\frac{1}{2^n};1) \right \}$ và hàm trên $D_n$: $h_n(X)=\bigcup_{A\subset X}f(X),A\subset H$. Ta chứng minh được $h_n$ tăng, theo bổ đề 3 ta được các tập $X_1,X_2,...$ sao cho $h_k(X_k)=X_k$. Ta cũng chứng minh được $X_{k+1}\subset X_k$. Theo bổ đề về dãy các đoạn thẳng lồng nhau, ta chứng minh được tồn tại tập $X$ sao cho $f(X)=X$.

Tại em không biết trình bày mấy cái này, nên em ghi tắt (chắc em cũng lập luận sai ở đâu đó).

Anh học mấy cái này ở đâu vậy?




#658657 Chứng minh rằng tồn tại $f(A)=H \setminus B,g(B)= H \setmi...

Đã gửi bởi redfox on 21-10-2016 - 10:50 trong Các dạng toán khác

Không liên quan là sao ạ. Em chứng minh bổ đề 3 với tập không đếm được mà.



#658636 Chứng minh rằng tồn tại $f(A)=H \setminus B,g(B)= H \setmi...

Đã gửi bởi redfox on 20-10-2016 - 22:28 trong Các dạng toán khác

Ta có các bổ đề sau

Bổ đề 1: $A\subset B\Rightarrow A\cap C\subset B\cap C, A\setminus C\subset B\setminus C$.

Bổ để 2: $f(A\cap B)\subset f(A)\cap f(B)$.

Bổ đề 3: Nếu $f$ tăng trên $\rho (H)$ thì $f$ luôn có một điểm bất động.

Chứng minh: Ta chứng minh bằng quy nạp với số phần tử của $H$

$\left | H \right |=0$, hiển nhiên.

Giả sử với $\left | H \right |\leq n$ bài toán đúng. Xét tập $H'=H\cup \left \{ a \right \},H\cap \left \{ a \right \}=\varnothing ,\left | H \right |=n$. Xét hai hàm trên $\rho (H)$: $g(A)=f(A)\setminus \left \{ a \right \},h(A)=f(A\cup \left \{ a \right \})\setminus \left \{ a \right \}$. Theo bổ đề 1, hai hàm này tăng, do vậy theo giả thiết quy nạp tồn tại hai tập $A,B\subset H$ sao cho $g(A)=A, h(B)=B$.

Nếu $f(B\cup \left \{ a \right \})=B\cup \left \{ a \right \}$, bài toán đúng với $n+1$.

Nếu $f(B\cup \left \{ a \right \})=B$, xét tập $A\cap B\setminus H$, theo bổ đề 2 ta có $\forall X\subset A\cap B, f(X)\subset A\cap B$, do vậy theo giả thiết quy nạp bài toán đúng với $n+1$.

Bổ đề $4$: $A\subset B\Leftrightarrow C\setminus B\subset C\setminus A$.

Ta quay lại bài toán. Xét hàm $h(X)=f(H\setminus g(H\setminus X))$. Theo bổ đề 4 ta có hàm $h$ tăng. Theo bổ đề 3 ta có $h$ tồn tại điểm bất động $X$. Dễ thấy hai tập $H\setminus g(H\setminus X) ,H\setminus X$ thỏa mãn đề bài.

(Q.E.D)

Bài này lạ quá. Anh lấy ở đâu vậy.




#668494 Đề thi chọn đội dự tuyển lớp 10 PTNK - ĐHQGTPHCM

Đã gửi bởi redfox on 15-01-2017 - 23:31 trong Thi HSG cấp Tỉnh, Thành phố. Olympic 30-4. Đề thi và kiểm tra đội tuyển các cấp.

Kí hiệu sai thì phải $A\oplus B=(A\setminus B)\cup(B\setminus A)$

Đúng rồi đó:https://en.wikipedia...tric_difference

Có nhiều kí hiệu




#668492 Đề thi chọn đội dự tuyển lớp 10 PTNK - ĐHQGTPHCM

Đã gửi bởi redfox on 15-01-2017 - 22:57 trong Thi HSG cấp Tỉnh, Thành phố. Olympic 30-4. Đề thi và kiểm tra đội tuyển các cấp.

Câu 4:

Ta sẽ chứng minh quy nạp theo $m$ cho điều này. Trước hết ta phát biểu bổ đề: $A\Delta B=A\Delta C$ thì $B=C$.

Giả sử $B\neq C$, khi đó không giảm tổng quát giả sử giả sử $a$ là phần tử thỏa $a\in B$ và $a\notin C$. Ta có hai trường hợp như sau:

   Case 1: Nếu $a\in A$ khi đó $a\notin (A\setminus B),a\notin (B\setminus A)\Rightarrow a\notin A\Delta B$. Nhưng lại có $a\in (A\setminus C)$ nên suy ra $a\in A\Delta C$ nên $A\Delta B\neq A\Delta C$, vô lý.

   Case 2: Nếu $a\notin A$ thì chứng minh tương tự suy ra $a\notin A\Delta C$ và $a\in A\Delta B$ nên suy ra $A\Delta B\neq A\Delta C$. Như vậy ta suy ra $B=C$.

Bây giờ ta sẽ quy nạp theo $m$. Với $m=1$ thì ta có một tập thuộc $T$ là tập rỗng. Với $m=2$ và hai tập $A,B$ thì ta có hai tập thuộc $T$ là tập rỗng và $A\Delta B$ thỏa. Như vậy giả thiết đúng với $m=1,2$.

Giả sử giả thiết đúng với $m=k$ thì ta chứng minh nó đúng với $m=k+1$. Xét $m+1$ tập $A_1,A_2,...,A_{m+1}$. Nếu với $m$ tập $A_1,A_2,...,A_m$ mà số lượng tập tạo thành không nhỏ hơn $m+1$ thì khi đó ta thêm vào một tập $A_{m+1}$ thì giả thiết vẫn đúng. Do đó ta chỉ xét cho trường hợp $|T|=m$.

Khi đó, nếu ta thêm vào một tập $A_{m+1}$ thì ta sẽ thêm vào tập $T$ các tập hợp $A_{m+1}\Delta A_1,...,A_{m+1}\Delta A_{m+1}$. Nếu các tập này trùng với $m$ tập đã có trong $T$ thì do $|T|=m$ nên theo nguyên lý Dirichlet tồn tại $i,j,1\leq i<j\leq m+1$ để $A_{m+1}\Delta A_i=A_{m+1}\Delta A_j$ và theo bổ đề ta có $A_i=A_j$, vô lý. Vậy trong $m+1$ tập đó chắc chắn có một tập khác với các tập trong $T$ và số phần tử của $T$ tăng lên ít nhất một đơn vị, tức là $|T|\geq m+1$.

Vậy giả thiết quy nạp là đúng và ta có điều phải chứng minh.

 

P.S: Cách giải bài số 2 của mình tương đối giống nhưng hơi khác so với cách của anh Lâm Hữu Phúc, mình sẽ đăng sau

Lấy luôn $S=\left \{ A_1\bigtriangleup A_k|1\leq k\leq m \right \},\left | S \right |=m,S\subset T$




#658237 Đề chọn đội tuyển học sinh quốc gia thành phố Hồ Chí Minh năm 2016-2017 (ngày 2)

Đã gửi bởi redfox on 17-10-2016 - 22:11 trong Thi HSG cấp Tỉnh, Thành phố. Olympic 30-4. Đề thi và kiểm tra đội tuyển các cấp.

Giả sử $1\in A$. Ghi các số tự nhiên từ $1$ đến $3n$, đánh các chữ $a,b,c$ tương ứng với các tập các số đó thuộc.

-Hai chữ $b,c$ liên tiếp: Hai số đó với $1$ thỏa mãn.

-Các chữ $b,c$ bị ngăn cách bởi $a$. Giả sử không tồn tại chuỗi $bba$. Khi đó xét các chữ $a$ cần dùng để ngăn cách các chữ $b$, ta có $\left | A \right |>\left | B \right |$ (vô lý). Vậy tồn tại chuỗi $bba$, tương tự tồn tại chuỗi $acc$. Lấy các số $x,y$ tương ứng với chữ $b$ thứ $2$ và chữ $c$ thứ $1$. Giả sử $x>y$ (TH còn lại trừ ngược) ta có $x-y=(x+1)-(y+1)=(x-1)-(y-1)$ và $x,x-1\in B,y,y+1\in C,x+1,y-1\in A$. Xét chữ được đánh dấu trên hiệu $x-y$, ta luôn có $3$ số thỏa mãn.

(Q.E.D)




#696820 Đề thi chọn đội tuyển tp Đà Nẵng

Đã gửi bởi redfox on 19-11-2017 - 15:22 trong Thi HSG cấp Tỉnh, Thành phố. Olympic 30-4. Đề thi và kiểm tra đội tuyển các cấp.

Lời giải của bạn chưa chính xác. Phép quy nạp của bạn chỉ giúp chỉ ra là $n-1$ thỏa mãn chứ ko chứng minh nó lớn nhất.

Có thể chuyển điều kiện bài toán thành như thế này.

Tìm $k$ lớn nhất để mà 2 tập bất kì trong $k$ tập này thỏa mãn 3 điều kiện sau:

1) Không có tập nào là $S_n$

2) 2 tập này ko là phần bù của nhau

3) 2 tập này hoặc rời nhau hoặc là có tập này chứa tập kia

 

Vd tập các tập hợp này thỏa mãn $\left \{1 \right \},\left\{2 \right \},...,\left \{2016 \right \},\left \{2017 \right \},\left \{1,2 \right \},\left \{1,2,3 \right \},...,\left \{1,2,3,..,2015 \right \}$.

mình đã chứng minh $k_n \leq n-1$ rồi bạn.




#693591 Đề thi chọn đội tuyển tp Đà Nẵng

Đã gửi bởi redfox on 23-09-2017 - 20:44 trong Thi HSG cấp Tỉnh, Thành phố. Olympic 30-4. Đề thi và kiểm tra đội tuyển các cấp.

Bài 3 ngày 2: Tổng quát cho $n>1$. Đặt $S_i=\left \{ 1,...i \right \}$. Xét $k_n$ lớn nhất sao cho tồn tại $k_n$ tập con của $S_n$ thỏa mãn.

Ta có $S_1,...S_{n-1}$ thỏa mãn nên có thể giả sử $k_n\geq n-1$. Ta sẽ chứng minh $k_n=n-1$ bằng quy nạp

Với $n=2$ dễ có $k_2=1$

Với $n\geq 3$.Ta có $A_i\neq \varnothing ,S_{n}$. Ta có $A$ và $A'$ không xuất hiện cùng nhau trong $k$ tập trên và có thể thay $A$ bằng $A'$ trong $k_n$ tập trên mà vẫn thỏa mãn điều kiện, vậy ta có thể thay tất cả các tập không chứa $n$ như trên, vậy ta có thể giả sử $k_n$ tập trên đều chứa $n$. Nếu trong $k_n$ tập trên không chứa tập $\left \{ n \right \}$ thì ta vẫn có thể thêm tập đó vào mà vẫn thỏa mãn điều kiện, thu được $k_n+1$ tập thỏa mãn trái với cách chọn $k_n$. Vậy trong $k_n$ tập trên chứa $\left \{ n \right \}$. Bỏ tập $\left \{ n \right \}$ và bỏ phần tử $n$ trong các tập còn lại thu được $k_n-1$ tập con của $S_{n-1}$ thỏa mãn. Ta có$k_n-1\leq k_{n-1}= n-2\Rightarrow k_n\leq n-1$ (theo giả thiết quy nạp và cách chọn $k_{n-1}$). Từ trên ta có $k_n=n-1$.

Cụ thể với bài trên, $k= k_{2017}= 2016$.




#653431 Đề thi chọn đội tuyển chuyên Thái Bình

Đã gửi bởi redfox on 08-09-2016 - 22:38 trong Thi HSG cấp Tỉnh, Thành phố. Olympic 30-4. Đề thi và kiểm tra đội tuyển các cấp.

Bài 3: Thay đổi vai trò của $x, y$ ta có $\left | f(x)-f(y)+g(y)(x-y) \right |\leq M\left | x-y \right |^{2+a} \Rightarrow \left | (x-y)(g(x)-g(y)) \right |\leq \left | f(x)-f(y)+g(y)(x-y) \right |+\left | f(y)-f(x)-g(x)(x-y) \right |\leq 2M\left | x-y \right |^{2+a} \Rightarrow \left | g(x)-g(y) \right |\leq 2M\left | x-y \right |^{1+a}$.

Với $x>y$, $n$ là số nguyên dương tuỳ ý, chọn $\partial = \frac{x-y}{n}$. Ta có:

$\left | g(x)-g(y) \right |\leq \sum_{k=0}^{n-1}\left | g(x+k\partial )-g(x+(k+1)\partial ) \right |\leq Mn\partial ^{1+a}=M(x-y)\partial ^a$

Lấy giới hạn ta dễ có $\left | g(x)-g(y) \right |=0$ suy ra $g(x)$ là hàm hằng: $g(x)=C$.

Vậy $\left | (f(y)-Cy)-(f(x)-Cx) \right |\leq M\left | x-y \right |^{2+a}$. Chứng minh tương tự ta có $f(x)-Cx$ là hàm hằng: $f(x)=Cx+D$

($C, D$ là hằng số)




#649431 Trường hè toán học 2016 bài kiểm tra số 1

Đã gửi bởi redfox on 13-08-2016 - 18:27 trong Thi HSG cấp Tỉnh, Thành phố. Olympic 30-4. Đề thi và kiểm tra đội tuyển các cấp.

4b)Không.
Cột thứ $i$ chứa $10-i$ ô đỏ nên tổng số ô đỏ là $45$ không chia hết cho $10$.



#647565 Trường hè toán học năm 2016 (phần đại số)

Đã gửi bởi redfox on 01-08-2016 - 22:22 trong Thi HSG cấp Tỉnh, Thành phố. Olympic 30-4. Đề thi và kiểm tra đội tuyển các cấp.

Bài 2

Giả sử các số âm, số dương trong $n$ số đã cho lần lượt là $b_1;b_2;...;b_m, c_1;c_2;...;c_p$. Giả sử $m\leq p\Rightarrow m\leq \left \lfloor \frac{n}{2} \right \rfloor$. Vì $b_i\in (-1;0)$ nên $-\sum_{i=1}^{m}b_i=\sum_{i=1}^{p}c_i<m$; $b_i^2<-b_i$. Tương tự $c_i^2<c_i$. Vậy

$2k=\sum_{i=1}^{n}a_i^2=\sum_{i=1}^{m}b_i^2+\sum_{i=1}^{p}c_i^2<-\sum_{i=1}^{m}b_i+\sum_{i=1}^{p}c_i<2m\leq 2\left \lfloor \frac{n}{2} \right \rfloor\Rightarrow n\geq 2k+2$

Với $n=2k+2$ ta chọn $k+1$ số bằng $-\sqrt{\frac{k}{k+1}}$, $k+1$ số bằng $\sqrt{\frac{k}{k+1}}$ thỏa mãn. Vậy $n=2k+2$